Tài liệu hạn chế xem trước, để xem đầy đủ mời bạn chọn Tải xuống
1
/ 28 trang
THÔNG TIN TÀI LIỆU
Thông tin cơ bản
Định dạng
Số trang
28
Dung lượng
304,89 KB
Nội dung
PHÂNHOẠCHNGUYÊN ĐẶNG THU HƯƠNG THPT CHUYÊN HẠ LONG Email: dangthuhuong91@gmail.com Tóm tắt nội dung Thế gọi "số học-tổ hợp" Thực khơng có định nghĩa cho loại tốn đó, hiểu thực chất tốn tổ hợp làm hấp dẫn thêm vài tính chất số học số tự nhiên Những toán dạng "số học-tổ hợp" coi nghiên cứu "cấu trúc tập số tự nhiên" thay nghiên cứu mối liên hệ đại số tính chất không rời rạc số tự nhiên mơn đại số, giải tích hay số học khác Chủ đề "số học-tổ hợp" rộng toán học, bao gồm kiến thức tổ hợp số học, chí đại số cao cấp Tuy nhiên mối quan tâm dành cho dạng toán giới hạn Để tiện theo dõi, tơi tạm chia đề tài thành phần sau: Phânhoạch tập hợp số nguyên, Phânhoạch vấn đề liên quan, Tính chia hết Các tốn tổng Trong đề tài tơi trình bày vài mảng lớn thông dụng "số học- tổ hợp" thường gặp kỳ thi học sinh giỏi tất chủ đề "số học- tổ hợp" Mục lục Phânhoạch tập hợp số nguyênPhânhoạch vấn đề liên quan Tính chia hết 17 Các tốn tổng 22 §1 Phânhoạch tập hợp số nguyên Một mục quan trọng số học tổ hợp nghiên cứu phânhoạch tập hợp số nguyên Ví dụ, kết Schur nêu rằng: "Mọi số nguyên dương k, tồn số nguyên n thỏa mãn: số nguyên từ 1, 2, , n chia thành k tập con, tập chứa số phân biệt cho có số tổng hai số kia" Hay định lý Van der Waerden sau: "Nếu tập tất số nguyên dương chia hữu hạn tập con, tồn tập chứa dãy cấp số cộng dài tùy ý" Để tìm thêm tính chất mà tập phânhoạch ngun có, ta cần tìm hiểu thêm mối quan hệ tập Phần ta giới hạn mục phânhoạch tập hợp số nguyên kỹ thuật xây dựng, quy nạp, phản chứng, chứng minh trực tiếp sử dụng tập Bài tốn 1: Có thể hay khơng số 1, 2, , 100 lập lên đủ 12 dãy cấp số nhân? (Russia, 1995) Lời giải: Một tập hợp dãy số {(ai,n )n∈N }i∈I gọi phủ kín tập S S ⊆ i∈I n∈N {ai,n } Ta cố định i ∈ I, dãy (ai,n )n∈N gọi dãy cấp số nhân tồn số thực qi thỏa mãn ai,n = ai,0 qin với n ∈ N Giả sử phản chứng, tồn ba số nguyên tố phân biệt p1 , p2 , p3 thỏa mãn {p1 , p2 , p3 } ⊆ n∈N {ai,n } ≤ Nếu |qi | > n∈N {ai,n } với i ∈ I Nếu |qi | ∈ {0, 1} (tương tự với trường hợp < |qi | < 1, ta có p1 < p2 < p3 ) Suy tồn số nguyên dương x1 < x2 < x3 thỏa mãn x pj = ai,xj = ai,0 qi j với j ∈ {1, 2, 3} Đặc biệt qi = x2 −x1 p2 p1 = p3 p2 x3 −x2 , rõ ràng xảy Tức dãy cấp số nhân chứa nhiều số nguyên tố phân biệt Đặc biệt kéo theo S ⊆ i∈I n∈N {ai,n } với tập S xác định bất đẳng thức sau đúng: |S ∩ P| ≤ i∈I n∈N {ai,n } ∩ P ≤ 2|I| Mệnh đề phủ định dẫn đến bất đẳng thức sai π(100) ≤ 24 (có 25 số nguyên tố từ đến 100), đặt S = {1, , 100} I = {1, , 12} Từ toán ta nhận thấy dãy 1, 2, , 100 khơng thể phủ kín 12 dãy cấp số nhân, 12 dãy có phải điều kiện chặt khơng? Ta xét tốn sau: Bài tốn 2: Có thể hay khơng dãy 2, 3, , 100 phủ kín 20 dãy cấp số nhân? Lời giải: Ta chứng minh câu trả lời khơng, rõ ràng tốn mạnh toán ban đầu Cố định i ∈ I giả sử (ai,n )n∈N dãy hình học thỏa mãn n∈N {ai,n } ∩ P ≥ Khi ta có kết sau n∈N {ai,n } ∩ Z = 2, kết mạnh Theo chứng minh tốn ban đầu, ta có n∈N {ai,n } ∩ P = Ta phải chứng minh khơng tồn số ngun khác phủ kín dãy số Xác đinh số nguyên x1 < x2 trước thỏa mãn p1 = ai,x1 = ai,0 qix1 p2 = ai,x2 = ai,0 qix2 với số nguyên tố p1 < p2 số thực qi lớn Khi qi = (p2 /p1 ) x2 −x1 , phần tử dãy cấp số nhân có dạng định, suy n∈N {ai,n } = p1 z∈Z∩[−x1 ,∞) p2 p1 z x2 −x1 Giả sử với ngược lại tồn số nguyên α ∈ {p1 , p2 } y ≥ −x1 thỏa mãn ai,y = α Khi h x −x1 p2 α= h −1 x −x1 p1 Rõ ràng điều kiện y ∈ {p1 , p2 } tương đương với h ∈ {0, x2 − x1 } Nếu h lớn x2 − x1 phân số tối giản, khơng số nguyên Nếu < h < x2 − x1 α có dạng pl1 pm với số hữu tỉ < l, m < tương ứng, khơng |h| x −x +1 |h| x −x số nguyên Cuối cùng, h âm, α có dạng p1 /p2 , số nguyên Xét số i ∈ I thỏa mãn n∈N {ai,n } ∩ P = {p}, với số nguyên tố p Và giả sử n∈N {ai,n } ∩ Z ≥ Khi tồn số nguyên β = p lớn thỏa mãn ai,x1 = β ai,x2 = p với x1 , x2 số nguyên không âm Nó kéo theo qi = (β/p) x2 −x1 , số hạng z dãy có dạng p(β/p) x2 −x1 với số nguyên z ≥ −x2 Mặt khác, giả sử β = pk với số nguyên k ≥ Khi hoạng tử dãy có z dạng pk x2 −x1 , số nguyên x2 − x1 | z Điều có nghĩa tập hợp giao với Z tạo bời phần tử có dạng pkn với số ngun khơng âm n z Mặt khác, giả sử p ∤ β Khi υp p(β/p) x2 −x1 phải thuộc N, điều xảy z ∈ {0, x2 − x1 } Vậy kết luận khơng có số nguyên khác bị phủ trường hợp Để tính tổng trường hợp tốt nhất, dãy cấp số nhân (2n )n≥1 (3 · 2n )n≥1 phủ số nguyên {2, 3, , 100} Tương tự, (5 · 2n )n≥1 (4 · 2n )n≥1 phủ nhiều số nguyên, dãy (4 · 2n )n≥1 dùng chung với dãy (2n )n≥1 Tất số phủ nhiều số nguyênphân biệt (*) Gọi A số dãy thỏa mãn dãy phủ nhiều số nguyên tố, B số dãy số mà dãy phủ số ngun tố (đặc biệt có số nguyên số nguyên tố!) Khi A + 2B = 25 6A + 2B ≥ 99 Vẽ đồ thị A − B dễ thấy A + B có giá trị nhỏ 20 Chú ý 1.0.1 Số 20 dãy cấp số nhân chặn tốt 24 dãy cấp số nhân với nhận xét (*) Chính xác ta cần dùng 36 dãy cấp số nhân để phủ kín dãy số 1, 2, , 100 (đọc thêm [6]) Có nhiều cách phânhoạch sử dụng 36 dãy cấp số nhân để phủ kín 1, 2, , 100 Đây ví dụ phânhoạch sử dụng 36 dãy cấp số nhân sau: (1,2,4,8,16,32,64) (3,6,12,24,48,96) (5,10,20,40,80) (7,14,28,56) (9,18,36,72) (11,22,44,88) (13,26,52) (15,30,60) (17,34,68) (19,38,76) (21,42,84) (23,46,92) (25,35,49) (27,45,75) (50,70,98) (81,90,100) đôi không giao phủ + + + 3.4 + 10.3 = 60 số nguyên Sử dụng thêm 20 dãy cáp số nhân, dãy có hai phần tử để phủ kín 40 số ngun lại Vậy ta sử dụng 16 + 20 = 36 dãy cấp số nhân Bài tốn 3: Bạn muốn tơ màu cho số dãy từ đến 100 cho hai số có số ước số số tơ khác màu Hỏi số màu cần sử dụng bao nhiêu? Lời giải: Dễ thấy màu đáp số tốn 1, 2, 4, 8, 16, 32 64 phải tô màu khác Đó điều kiện đủ: n có m ước ngun tố phân tích thành nhân tử, n tô màu thứ m + Màu nhỏ ta sử dụng (cho 1) màu lớn ta sử dụng (với 64 96) Bài toán 4: Tập số nguyên dương phânhoạch hữu hạn tập Chứng minh với cách phânhoạch S thành tập S1 , S2 , , Sn có tính chất sau: với số ngun dương n ,tồn i cho Si chứa vô hạn bội n (BMC contest, 1999) Lời giải: Gọi tập S1 , S2 , , Sk Giả sử phản chứng ta chứng minh mâu thuẫn Khi với Si tồn ni thỏa mãn Si chứa hữu hạn bội ni Đặt n = n1 n2 nk ; Khi bội n bội ni tập Si chứa hữu hạn bội n Nhưng điều có nghĩa k tập chứa hữu hạn bội n, chúng phânhoạch tập hợp số nguyên dương (chứa vô hạn bội n), nên ta có mâu thuẫn Bài toán 5: Chứng minh tập số nguyên dương chia thành vơ hạn tập vô hạn cho : x, y, z, w thuộc tập con, x − y z − w thuộc z tập xy = w Lời giải: Đặt Ak tập chứa số có dạng (2k − 1)2n với n = 0, 1, 2, Ta chứng minh cách chia thỏa mãn yêu cầu toán Giả sử x, y, z, w ∈ Ak với x > y, z > w Ta có x = (2k − 1)2a+b , y = (2k − 1)2a , z = (2k−1)2c+d ; w = (2k−1)2c Khi x−y = (2k−1)(2b −1)2a ; z−w = (2k−1)(2d −1)2c z z Ta có xy = 2b ; w = 2d Suy xy = w ⇐⇒ b = d ⇐⇒ x − y z − w có ước lẻ lớn Bài tốn 6: Giả sử số nguyên dương tô bốn màu: đỏ, xanh, lục vàng Gọi m n số nguyên lẻ mà có giá trị tuyệt đối khác Chứng minh tồn số tô màu cho hiệu chúng giá trị sau : m, n, m − n, m + n (IMO Proposal, 1999) Lời giải: Cách 1: Giả sử ngược lại Tô màu cho lưới 2D cách (i, j) phải tô màu với mi+nj Bây ta chọn đơn vị vuông Các đỉnh (i, j), (i+1, j), (i, j+1), (i+1, j+ 1) với i, j bất kỳ, điểm tô màu với h, h+m, h+n, h+m+n , với h = mi + nj Giả sử có số h, h + m, h + n, h + m + n tô màu, hiệu chúng thuộc giá trị sau m, n, m + n m − n Mâu thuẫn Do đỉnh vuông đơn vị tô màu khác Giả sử có điểm liền kề hàng lưới tô màu khác Khơng tính tổng qt giả sử: R B G Ở hình vẽ , ∗ phải Y ta tơ hết điểm kề màu thích hợp Khi x phải B, tiếp tục Do ta mở rộng cột khơng xác định thấy chúng tuần hồn có chu kỳ theo hàng Khi dễ thấy hàng lưới 2D phải tuần hoàn với chu kỳ B G R B G Y R→G Y R x R B G Nhưng khơng có điểm kề hàng cho tô màu khác cột phải tuần hồn với chu kỳ 2, tất cột mạng lưới có chu kỳ Do hoạc tất hàng có chu kỳ 2, tất cột có chu kỳ R B ∗ G R →G Giả sử tất hàng có chu kỳ Khơng tính tổng qt giả sử (0, 0) R (1, 0) B Khi dễ thấy hàng , (0, 1), (1, 1), (2, 1), tô màu G Y , hàng , (0, 2), (1, 2), (2, 2), tô màu R B tiếp tục Do (0, m) phải G Y với m số lẻ Nhưng (n, 0) phải R B Nhưng số nguyên mn có màu với (0, m) (n, 0) Mâu thuẫn Tương tự tất cột có chu kỳ Vậy điều giả sử sai, hay tồn số tô màu cho hiệu chúng giá trị sau : m, n, m − n, m + n Cách 2: Dưới cách giải theo hướng số học cho toán! Giả sử x, y > (Rõ ràng ta làm x, y âm ta thay x −x và/hoặc y −y) Vì ta cần xét tốn với số ngun khơng âm Phủ định mệnh đề cần chứng minh có tập S ⊂ N thỏa mãn S, S + x, S + y, S + x + y tập dời nhau, hợp chúng N Điều có nghĩa s + + t + t2 + = (1 + tx )(1 + ty ) Giả sử y < x Ta có 1+t+t = s∈S t 1+ty y−1 2y 2y+1 3y−1 (1 + t + + t ) + (t + t + + t ) + = E Khi ta chia E cho + tx ta phải thu chuỗi có hệ số s (chính s∈S t ), điều dẫn tới mâu thuẫn sau: chuỗi ta y−1 chứa 1+t+ .+t , y < x nên phải tồn k thỏa mãn x ∈ (2ky, (2k +1)y −1] (do x lẻ) Tuy nhiên tồn α ∈ {1, 2, , y − 1} thỏa mãn x + α = (2k + 1)y α ∈ S (do x > y số dương nên {1, 2, , y − 1} ∈ S ) , điều có nghĩa s chuỗi s∈S t ) ta khơng có hệ số âm nào, nhân biểu thức với x (2k+1)y + t chứa t , E khơng có số hạng này, ta có mâu thuẫn Từ ta có điều phải chứng minh Bài tốn 7: Mỗi số ngun tơ hai màu đỏ xanh Biết tập hợp hữu hạn A gồm vài số nguyên liên tiếp, chênh lệch số số tơ màu đỏ số số tô màu xanh A không 1000 Chứng minh tồn tập hợp gồm 2000 số nguyên liên tiếp mà có xác 1000 số tơ màu đỏ 1000 số tô màu xanh Lời giải: Gọi ak hiệu số màu đỏ trừ số màu xanh nằm tập {k, k + 1, , k + 1999} Nếu ak = ta có đpcm Giả sử khơng có k thỏa mãn ak = 0, khơng tính tổng quát giả sử a1 > Ta thấy |ak − ak+1 | ≤ với k, khác đượcc xác định màu k k + 2000 Khi ak = 0, a1 > ak số chẵn kéo theo ak > với k Xét số từ đến 2002000 Sự khác số số đỏ số số xanh đoạn a1 + a2001 + a4001 + · · · + a20000001 , 1001 số hạng dương suy lớn 1000 Điều mâu thuẫn với giả thiết cho Ta xét toán tương tự dạng mức độ khó sau: Bài toán 8: Giả sử 1000 học sinh đứng thành vòng tròn Chứng minh tồn số nguyên k với 100 ≤ k ≤ 300 cho vòng tròn có nhóm gồm 2k học sinh kề nhau, mà số học sinh nữ k học sinh số học sinh nữ k học sinh cuối (IMO shortlist 2011) Lời giải: Số học sinh xếp thứ tự từ đến 1000 Đặt = học sinh thứ i nữ, = học sinh thứ i nam Chú ý số xét theo modulo 1000 n+2k−1 n+k−1 Đặt f (n, k) = i=n − i=n+k với số nguyên n k ∈ [100, 300] Nếu ta có f (n, k) = với n, k đó, ta có điều phải chứng minh Giả sử f (n, k) = với n, k Dễ thấy f (n + 1, k) − f (n, k) ∈ [−2, 2], f (n + 1, k) − f (n, k) = an+2k + an − 2an+k 1000 f (i, k) = với k, học sinh đếm k lần mang dấu dương Khi đó, i=1 k lần mang dấu âm Suy phải tồn n để f (n, 100) > f (n + 1, 100) < ngược lại, dãy f (i, 100) tất dương tất âm, điều Dễ thấy f (n, 100) = f (n + 1, 100) = −1 khơng f (n + 1, 100) − f (n, 100) > Suy an+100 = vaf an = an+200 = Suy ta có dãy GXBXG với B kí hiệu trai, G kí hiệu gái, X kí hiệu dãy 99 học sinh mà số học sinh nữ nhóm Xét f (n − 1, 101) Nếu an−1 = f (n − 1, 101) = 0, mâu thuẫn Suy an−1 = Tương tự, an+201 = từ f (n, 101) Bằng quy nạp, ta có an−k = với k ∈ [0, 200] Khi f (n − 200, 100) = 0, mâu thuẫn Vậy ta có điều phải chứng minh Bài toán 9: Tập số nguyênphânhoạch thành dãy cấp số cộng rời Chứng minh tồn hai dãy cấp số cộng mà có cơng sai (Sasha Schwartz) Lời giải: Giả sử tập số nguyên dương bị chia thành hữu hạn tập S1 , S2 , , Sk , tập cấp số cộng thỏa mãn chúng khơng có phần tử chung Gọi Với |z| < xét Si = {ai + ndi với n ∈ N ∪ {0}}, di = dj for i = j ∞ z n + + zn = n=1 n∈S1 zn n∈Sk tương đương với, z a1 z ak z = + + d 1−z 1−z − z dk Do di = dj , ta giả sử d1 > dj for j = cho z → e2πi/d1 hạng z a1 tổng 1−z d1 tiến tới vô tất số hạng lại tổng z hiển nhiên có giới hạn hữu hạn Do ta có có giới hạn hữu hạn vế trái 1−z mâu thuẫn thu điều phải chứng minh Chú ý: Qua cách chứng minh này, ta khẳng định tập số nguyênphânhoạch thành cấp số cộng rời tồn dãy cấp số cộng có cơng sai cơng sai cơng sai lớn tất công sai cấp số cộng Bài tốn 10: Tìm số k ngun dương lớn có tính chất sau: tập hợp số nguyên dương phânhoạch thành k tập A1 , A2 , , Ak cho với n ≥ 15, với i ∈ {1, 2, , k} tồn hai phần tử khác Ai có tổng n Lời giải: Ta bắt đầu với trường hợp k = Rõ ràng phânhoạch tập hợp số nguyên dương thành hai tập A1 = {2n; n ≥ 3} ∪ {1, 2}; A2 = {2n − 1; n ≥ 3} ∪ {3, 4} có tính chất: số nguyên dương ≥ biểu diễn dạng tổng hai số thuộc A1 tổng hai số thuộc A2 Khi k = 3, dĩ nhiên việc phânhoạch thành"chẵn, lẻ" thay phânhoạch theo modulo 3, trước cần thêm vào lớp đồng dư modulo số số để bảo đảm tập biểu diễn số lớn 15 dạng tổng hai số Có thể chọn phânhoạch sau đây: A1 = {1, 2, 3} ∪ {3m; m ≥ 4}, A2 = {4, 5, 6} ∪ {3m − 1, m ≥ 4}, A3 = {7, 8, 9} ∪ {3m − 2; m ≥ 4} Dễ thử lại rằng, phânhoạch thỏa mãn Hơn nhận thấy rằng, điều kiện biểu diễn số 15phânhoạch chặt, chẳng hạn số 14 biểu diễn dạng tổng hai số thuộc A2 hai số thuộc A3 Từ dự đốn k = giá trị lớn để tồn phânhoạch thỏa mãn Ta chứng minh dự đoán trên, nghĩa với k ≥ phânhoạch tập số tự nhiên thành k tập hợp thỏa mãn Rõ ràng với k ≥ mà tồn phânhoạch thỏa mãn, phânhoạch tồn với k = 4; cần lấy phânhoạch A1 , A2 , A3 , A4 ∪ A5 ∪ ∪ Ak ta phânhoạch gồm tập hợp thỏa mãn Như cần chứng minh tồn phânhoạch gồm tập hợp thỏa mãn Giả sử tồn phânhoạch vậy: A1 , A2 , A3 , A4 Như ta thấy ví dụ k = 2, k = 3, tập hợp Ai phải chứa số số tự nhiên Xét 10 số nhỏ mà tập hợp Ai phải biểu diễn được: 15,16, ,24 Mỗi số 10 số tổng số thuộc tập hợp B = {1, 2, 3, , 23} Như vậy, tập hợp Ai cần chứa số thuộc tập hợp B Do bốn tập Ai rời mà B có 23 phần tử nên phải tồn tập Aj chứa số thuộc B, giả sử số x1 , x2 , x3 , x4 , x5 Năm số biểu diễn 10 số số từ 15 đến 24, tức 10 số 10 tổng {xk +xl , k = l; ≤ k ≤ 15} Từ suy 15 + 16 + + 24 = 4(x1 + x2 + x3 + x4 + x5 ) số xi tham gia cặp số Đẳng thức cho ta mâu thuẫn tổng vế trái 195, vế phải chia hết cho Bài toán 11: Mỗi số tập N = {1, 2, 3, · · · , n − 1}, với n ≥ 3, tô hai màu đỏ đen cho : (i) i n − i tô màu (ii) Với j ∈ N nguyên tố với n, i |j − i| tô màu với i ∈ N, i = j Chứng minh tất số N tô màu Lời giải: Ta kí hiệu a ∼ b a b tô màu quan hệ tương đương Lấy j ∈ N thỏa mãn gcd(n, j) = Nếu a < b, j | b − a, ta có b = a + kj, với k số nguyên dương Khi đó, theo (ii), ta có b = a + kj ∼ a + (k − 1)j ∼ · · · ∼ a + j ∼ a, nên a ≡ b (mod j) kéo theo a ∼ b Với ≤ c < j ta thấy c ∼ j − c j − c ∼ n − (j − c) = (c + n) − j Nhưng (c + n) − j ≡ c + n (mod j), suy c ∼ j − c ∼ (c + n) − j ∼ c + n xét modulo j ta có c ∼ c + n ∼ c + 2n ∼ · · · ∼ c + (j − 1)n Nhưng gcd(n, j) = 1, nên modulo j phần tử c, c + n, c + 2n, , c + (j − 1)n hệ thặng dư đầy đủ, ta có hoán vị 0, 1, 2, , j − 1, nên ∼ ∼ ∼ · · · ∼ j − 1, suy tất số N tô màu Bài toán 12: Tập số nguyên dương chia hữu hạn tập rời Chứng minh tồn tập ,giả sử Ai , có tính chất sau: Tồn số ngun dương m thỏa mãn với k, ta tìm số a1 , a2 , , ak thuộc Ai với < aj+1 − aj ≤ m cho j, ≤ j ≤ k − (IMO Proposal, 1990) Lời giải: Gọi tập A ⊂ N tốt thỏa mãn yêu cầu toán nghĩa tồn m thỏa mãn với k tồn a1 , a2 , , ak ∈ A thỏa mãn < ai+1 − ≤ m, với i = 1, 2, , k − Gọi dãy bị chặn x tăng ngặt số hạng khác liên tiếp sai khác khơng q x Bổ đề Nếu A, B ⊂ N thỏa mãn A ∩ B = ∅ A ∪ B tốt, có tập A B tốt Do A ∪ B tập tốt nên tồn m cho thỏa mãn đề Nếu A tập tốt Vậy với m tồn k(m) cho với k(m) số thuộc A tồn số liên tiếp m Ta chứng minh tập B tập tốt Do A ∪ B tôt nên xét k = (k(m) + 1)2 tồn a1 < a2 < · · · < ak phần tử thuộc A ∪ B mà hiệu số liên tiếp nhỏ m Do dãy dãy tốt nên có không k(m) phần tử thuộc A tồn dãy B có độ dài k(m) cho hiệu hai số liên tiếp nhỏ m (kể độ dài lớn k(m)) Vậy B tập tốt Quay trở lại toán, gọi A1 ∪ A2 ∪ ∪ An phânhoạch tập số nguyên Nếu A1 tốt, ta có điều cần tìm Nếu A1 khơng tốt, A2 ∪ A3 ∪ ∪ An tốt Nếu A2 tốt tốn giải A2 khơng tốt, A3 ∪ ∪ An tốt, tiếp tục Cuối ta An−1 khơng tốt, An tốt Bài tốn 13: Xác đinh có tồn hay không số nguyên n > thỏa mãn: Tập số nguyên dương phânhoạch thành n tập khác rỗng cho tổng n − phần tử mà lấy phần tử từ tập n − tập con, thuộc tập lại (IMO Proposal, 1995) Lời giải: Trả lời: khơng Rõ ràng khơng có phânhoạch với trường hợp n = Xét n > Gọi phânhoạch thỏa mãn đề A1 ∪ A2 ∪ · · · ∪ An Ta chứng minh a, b, c (không thiết phân biệt) thuộc tập a + b a + c phải thuộc tập Giả sử a, b, c thuộc A1 , a + b thuộc A2 a + c thuộc A3 Lấy phần tử a4 thuộc A4 , a5 thuộc A5 , , an thuộc An Khi c + (a + b) + a4 + · · · + an phải thuộc A3 Nhưng b + (a + c) + a4 + · · · + an , tổng phải thuộc A2 mâu thuẫn Giả sử a + b thuộc A1 a + c thuộc A3 Như trước ta lấy thuộc Ai Nên b+a3 +· · ·+an phải thuộc A2 , (b+a3 + +an )+(a+c)+a4 +· · ·+an = (a+b+ c)+(a3 +· · ·+an )+(a4 +· · ·+an ) phải thuộc A1 Nhưng c+a3 +· · ·+an phải thuộc A2 , (a+b)+(c+a3 +· · ·+an )+a4 +· · ·+an = (a+b+c)+(a3 +· · ·+an )+(a4 +· · ·+an ) phải thuộc A3 Mâu thuẫn Chỉ lại trường hợp a + b a + c thuộc tập con.(*) Lấy từ Ai Đặt s = a1 + · · · + an Khơng tính tổng qt giả sử s thuộc A1 Đặt bi = s − Ta biểu diễn bi đưới dạng tổng phần tử từ tập trừ Ai , nên bi phải thuộc Ai Ta có + bi = s thuộc A1 Suy + = 2ai thuộc A1 (theo kết (*)) Giả sử 2m số dương chẵn Ta có m thuộc Ai Lấy tập bj , thỏa mãn bj = aj trừ bi = m Lập luận tương tự 2b1 , 2b2 , , 2bn thuộc tập Nhưng ta lấy j khác i, ta có 2bj = 2aj thuộc A1 Nên 2m phải thuộc A1 Nói cách khác, tất số chẵn thuộc A1 Xét a1 + a3 + · · · + an Tổng thuộc A2 Giữ a3 , a4 , , an cố định thay a1 số chẵn Khi ta thấy tất số lẻ lớn a3 + + an phải thuộc A2 Tương tự cách xét a1 + a2 + a4 + + an , ta thấy tất số lẻ lớn a2 + a4 + + an phải thuộc A3 Mâu thuẫn Ta có điều phải chứng minh §2 Phânhoạch vấn đề liên quan Xét đồng thức A = B, với A B cách đếm khác tập hợp đối tượng Ta có đồng thức vậy? Một cách tiếp cận đếm số phần tử A chứng minh có số phần tử B Một cách tiếp cận khác tìm song ánh hai tập hợp A B Một ví dụ truyền thống cho thấy tương phản cách làm sau: số người số ghế hội trường Với cách thứ nhất, ta đếm số người phòng sau đếm số ghế phòng Tuy nhiên, theo cách thứ 2, ta cần cho người ngồi xuống đếm số ghế thừa số người khơng có ghế ngồi Khi chứng minh đẳng thức có liên quan đến phânhoach số nguyên, có dễ tiếp cận với cách thứ (hàm sinh), có lại dễ tiếp cận với phương án thứ hai (chứng minh song ánh), có dễ khó với hai phương án Trong viết ta xét ví dụ xảy theo ba trường hợp nói Trước hết ta cần tìm hiểu khái niệm phânhoạch số nguyên dương Định nghĩa 2.1 Nếu n số nguyên dương, phânhoạch n dãy số nguyên dương không tăng p1 , p2 , , pk có tổng n Khi pi gọi thành phầnphânhoạch Ta định nghĩa hàm p(n) số phânhoạch n Ví dụ p(5) = ta có phânhoạch n = 5: = = = = = = = 4+1 3+2 3+1+1 2+2+1 2+1+1+1 1+1+1+1+1+1+1 Ta có p(n) = với n số nguyên âm quy ước p(0) = Phânhoạchnguyên đề xuất Euler Sau nhiều năm với nhiều câu hỏi hay khó xác định tính chất giới hạn hay gọi tiệm cận p(n) với n đủ lớn Vấn đề cuối giải gần hoàn chỉnh Hardy, Ramanujan Rademacher [3] [4] Tuy nhiên câu hỏi sau: "Tìm tiêu chuẩn để xác định p(n) số lẻ hay chẵn?" cậu hỏi chưa giải đáp tính giá trị p(n) với giá trị n đến vài tỉ Các biểu đồ Ferrers phân chia số nguyên công cụ hữu ích để hình dung phân chia chúng minh vài đẳng thức liên quan đến phânhoạch số nguyên dương Biểu đồ Ferrers tương ứng với cách phânhoạch λ biểu diễn hình vẽ phânhoạch λ Để xây dựng biểu đồ Ferrers phânhoạch λ = (λ1 , λ2 , , λk ), đơn giản ta đặt hàng λi+1 bên hàng λi hàng trái, với i = 1, 2, , k − Ví dụ biểu đồ Ferrers phânhoạch λ = (6, 4, 3, 1, 1) sau Các cơng cụ sử dụng để đếm số lượng phânhoạch dựa vào dãy cấp số nhân: = + q + q2 + q3 + · · · 1−q = + q n + q 2n + q 3n + · · · − qn Chúng ta hình dung dãy cấp số nhân theo biểu đổ Ferrers theo hàng sau: 1− =1+ + + + ··· 1− + ··· 1− =1+ + + + ··· theo cột sau: 1− =1+ + + =1+ + + +··· Sau ví dụ sử dụng biểu đồ Ferrers chứng minh đẳng thức liên quan đến phânhoạch số tự nhiên Bài toán 1: Số phânhoạch số nguyên n có thành phần lớn k số phânhoạch n có k thành phần 10 Bài toán 7: Quy ước P (n) số cách phânhoạch n thành tổng số nguyên dương đặt Q(n) số cách phânhoạch 2n thành tổng số ngun dương cho khơng có xác n phần tử Chứng minh n−1 k=0 Q(2k ) = P (2n ) − Lời giải: Chúng ta chứng minh quy nạp Với n = ta chứng minh Q(1) = P (2)−1 Rõ ràng đúng, Q(1) = P (2) = Giả sử đẳng thức cần chứng minh với n, ta cần chứng minh đẳng thức với n + nghĩa cần chứng minh n k=0 Q(2k ) = P (2n+1 ) − Trước hết ta chứng minh điều sau: Q(2n ) = P (2n+1 ) − P (2n ) Ta nhận thấy P (2n+1 ) đếm tất phânhoạch 2n+1 thành thành phầnnguyên dương Mặt khác, Q(2n+1 ) đếm tất phânhoạch 2n+1 khơng có xác 2n thành phần Cho nên ta cần phải chứng minh số phânhoạch 2n+1 xác 2n thành phầnnguyên dương P (2n ) Điều hồn tồn tồn song ánh tập phânhoạch cách cộng trừ vào thành phần ý phânhoạch tính P (2n ) có tối đa 2n thành phần Ví dụ phânhoạch = + + + thành phần tương ứng với phânhoạch = + + + Ta có điều phải chứng minh Bài tốn 8: Với số nguyên dương m, ký hiệu C(m) số nguyên dương k lớn để tồn tập hợp S gồm m số nguyên cho số nguyên từ đến k thuộc S, tổng hai số thuộc S (không thiết khác nhau) Ví dụ: C(3) = 8; S = 1, 3, Chứng minh bất đẳng thức sau m(m + 3) m(m + 6) ≤ C(m) ≤ Lời giải: Điều kiện toán gợi cho ta thấy cần phải tính số phần tử tập hợp S ∪ (S + S), S + S = {x + y|x, y ∈ S} Nếu với tập hợp A, ký hiệu qua |A| số phần tử nó, ta có bất đẳng thức hiển nhiên sau: |S ∪ (S + S)| ≤ |S| + |S + S| Mỗi phần tử S + S nhận cách lấy tổng cặp số (bằng khác nhau) S, suy bất đẳng thức sau: |S + S| ≤ |S| + C|S| = |S|(|S| + 1) Trong hai bất đẳng thức cần chứng minh , bất đẳng thức bên phải tương đối dễ thấy Số C(m) số nguyên dương k lớn cho tồn tập S gồm m số nguyên dương thỏa mãn {1, 2, , k} ∈ S ∪ (S + S) Từ chứng minh ta có k ≤ |S ∪ (S + S)| ≤ |S|(|S| + 3) Từ suy m(m + 3) C(m) ≤ Ta chứng minh bất đẳng thức bên trái Trước tiên tập hợp S cần tìm chứa số số tự nhiên liên tiếp {1, 2, , t|t ≤ m} Với số này, tập hợp S ∪ (S + S) chứa số 1, 2, , 2t Như vậy, để biểu diễn số tiếp theo, cần thêm số 2t + Các số 1, 2, , t, 2t + cho phép biểu diễn số tự nhiên số 3t + 1, muốn biểu diễn số lớn hơn, ta cần thêm số 3t + Lý luận tập hợp S gồm m phần tử cần tìm có dạng {1, 2, , t} ∪ {(k + 1)t + k; k = 1, 2, , m − t} 14 Khi dễ thấy {1, 2, , m + (m + 1)t − t2 } ∈ S ∪ (S + S) Ta cần tìm giá trị t < m cho số số biểu diễn lớn nhất, tức tìm t cho m + (m + 1)t − t2 đạt cực đại Dễ chứng minh đại lượng đạt cực đại m+1 m(m + 6) t=[ ], giá trị cực đại [ ] Từ ta có điều phải chứng minh Bài tốn 9: Cho n số nguyên dương Giả sử ước nguyên dương phânhoạch thành cặp tức chia thành nhóm có phần tử thỏa mãn tổng cặp số nguyên tố Chứng minh số nguyên tố phân biệt khơng có số chúng ước n.(Benelux Mathematical Olympiad 2016) Lời giải: Gọi (da1 , db1 ), (da2 , db2 ), · · · , (dak , dbk ) tất ước nguyên dương nvà phânhoạch thỏa mãn đề Gọi dai + dbi = pci với ≤ i ≤ k Hiển nhiên pci ≥ số lẻ số dai , dbi phải số lẻ, số lại phải chẵn Điều dẫn đến n có xác k ước chẵn k ước lẻ Gọi n = 2r h n có xác τ (h) ước lẻ τ (n) = 2τ (h) Suy r = Xét ước nguyên tố lẻ p n mà pl |n ta có τ p có l · τ n pl n pl ước nguyên tố với ước mà chia hết cho p Rõ ràng xảy trường hợp tồn p > cho p | dai , dbi với cặp (dai , dbi ), có số chia hết cho p số không chia hết cho p Từ ta có l · τ n pl ≤τ n pl dẫn đến l = Vì n = 2p1 p2 · · · pm với m ∈ Z Từ ta nhận thấy dai = n dbi = 1, dai = 2p1 p2 · · · pm−1 dbi = pm (nó khơng thể ta biết cặp với n), , dai = 2p1 · · · pm−2 dbi = pm−1 pm (nó khơng thể số pm−1 , pm , số thuộc cặp trước rồi), Từ ta suy cặp (dai , dbi ) có dạng (k, n/k) với k ước is n Bây để chứng minh tốn ta giả sử phản chứng nghĩa tồn cặp khác (k, n/k) (l, n/l) thỏa mãn k + nk = l + nl = p suy (l − k)(n − lk) = 0, (k, n/k) = (l, n/l), mâu thuẫn Vậy số nguyên tố pci phải phân biệt Hiển nhiên gcd k + nk , n = gcd k, nk = (do n = 2p1 p2 · · · pm ) Nghĩa khơng có số ngun tố pci ước n Ta có điều phải chứng minh + Bài tốn 10: Tìm tất cách phânhoạch tập {1, 2, , n} thành tập A, B, C cho khơng có tập số tập sau A + B , B + C and C + A có chung phần tử Lời giải: Dễ thấy |X +Y | ≥ |X|+|Y |−1 Cho nên 2n−3 = |[3; 2n−1]| ≥ |A+B|+|B +C|+|C + A| ≥ (|A| + |B| − 1) + (|B| + |C| − 1) + (|C| + |A| − 1) = 2(|A| + |B| + |C|) − = 2n − Từ ta suy hợp tập A + B, B + C, C + A xác đoạn [3, 2n − 1] Khơng tính tổng quát , giả sử ∈ A, ∈ B Xét k phần tử bé C Giả sử k ≥ Nên số 3, , k nằm A + C B + C, thuộc A + B Do thuộc A + B nên ∈ B, phải thuộc A + B nên ∈ B, , tiếp tục [2, k − 1] ⊂ B Hơn biết k + = + k = + (k + 1) Do + k ∈ B + C, k + khơng thể thuộc B C + (k + 1) thuộc A + B A + C suy k + phải nằm tập chứa 1, tức A Do k + = + (k + 1) = + k, (B + C) ∩ (A + B) chứa + k, mà tập phải không giao , vô lý Vậy k = ∈ C Từ chứng minh quy nạp: l ≡ 1( mod 3), l ∈ A, l ≡ 2( mod 3), l ∈ B, l ≡ 0( mod 3), l ∈ C Quả thật mệnh đề với l ≤ Giả sử mệnh đề với l ≤ m Ta cần chứng minh mệnh đề với l = m + Ta có 2m−1 = m+(m−1) = (m+1)+(m−2) Chúng ta biết số m−2, m−1, m nằm tập khác tập A, B, C, m + phải thuộc tập có chứa m − 2, điều nghĩa mệnh đề chứng minh cho trường hợp l = m + Vậy có cách phânhoạch thỏa mãn đề phânhoạch theo số dư modulo 15 Bài toán 11: Cho Z Q tập số nguyên số hữu tỷ theo thứ tự a) Tồn hay không cách phânhoạch Z thành tập khác rỗng A, B, C cho tập A + B, B + C, C + A không giao nhau? b) Tồn hay không cách phânhoạch Q thành tập khác rỗng A, B, C cho tập A + B, B + C, C + A không giao nhau? Với X +Y ký hiệu tập {x +y : x ∈ X, y ∈ Y }, với X, Y ⊆ Z X, Y ⊆ Q.(IMO Shortlist 2012) Lời giải: (a) Câu trả lời có tồn phânhoạch Z thành tập khác rỗng A, B, C cho tập A + B, B + C, C + A không giao Gọi A = {x | x ≡ (mod 3)}, B = {x | x ≡ (mod 3)}, C = {x | x ≡ (mod 3)} Thì A+B = {x | x ≡ (mod 3)}, A+C = {x | x ≡ (mod 3)}, B+C = {x | x ≡ (mod 3)} tập không giao thỏa mãn đề Hơn nữa, chứng minh cách phânhoạch (khơng tính hốn vị chúng) Vì A, B, C tập khác rỗng, nên tồn a, b, c thỏa mãn a ∈ A, b ∈ B, c ∈ C Nếu a + ∈ A Thì (a + 1) + b ∈ A + B Nếu b + ∈ C, a + (b + 1) ∈ C, vơ lý Vậy b+1∈ / C Tương tự ta thu c + ∈ / B Nếu b + ∈ A c + ∈ A, b + (c + 1) ∈ A + B c + (b + 1) ∈ A + C, vô lý Nên hai điều sau xảy b + ∈ B, c + ∈ C Không tính tổng quát giả sử b + ∈ B Thì c + (b + 1) ∈ B + C Nếu c + ∈ A, b + (c + 1) ∈ A + B, vô lý Do c + ∈ / B, c + ∈ / A =⇒ c + ∈ C Tương tự ta có với k ∈ Z a + k ∈ A, b + k ∈ B, c + k ∈ C Giả sử b > a ta có a + (b − a) ∈ A, vô lý Tương tự cách chứng minh cho trường hợp a + ∈ A, với k ∈ Z ta có a + 2k ∈ A, b + 2k ∈ B, c + 2k ∈ C Theo định lý Dirichlet, tồn hai ba số a, b, c có tính chẵn lẻ Khơng tính tổng quát giả sử a, b chẵn Giả sử b > a a = 2x, b = 2y =⇒ 2x + 2(y − x) ∈ A, vô lý Nếu a + ∈ / A, không tính tổng quát giả sử a + ∈ B Khi a + c + ∈ B + C =⇒ c + ∈ A Do b + c + ∈ A + B, nghĩa b + ∈ C Khi a + c + ∈ A + B, a + b + ∈ B + C Vậy a + ∈ C Tương tự, c + ∈ B, b + ∈ A Tương tự ta có a + ∈ A, b + ∈ B, c + ∈ C Giả sử a + ∈ B a + + a + = a + a + ∈ A + B suy a + ∈ A, vô lý ta chứng minh Giả sử a + ∈ C a+1+a+2 = a+a+3 ∈ A+B suy a+2 ∈ C Tương tự ta lại có b+1 ∈ A, b+2 ∈ A, c + ∈ B, c + ∈ B Tuy nhiên (a + 2) + b ∈ C + B, (a + 1) + (b + 1) ∈ C + A Mâu thuẫn Cho nên với a ∈ A, ta có a + ∈ A, a + 1, a + ∈ / A Từ ta tồn cách phânhoạch tập số nguyên dương thỏa mãn đề b) Giả sử có phânhoạch Q tính chất thỏa mãn đề Ta chọn ba số hữu tỷ ri = pqii từ ba tập A, B, C, i = 1, 2, 3, tập N = 3q1 q2 q3 Cho S ⊂ Q tập số hữu tỷ với mẫu số N ( tối giản hay khơng) Tập tạo cách nhân số tự nhiên với số N1 , tập tập đóng phép cộng k phép trừ Hơn nữa, ta coi k ∈ Z phân số N ∈ S S coi tập số nguyên Z phép cộng Các số ri thuộc S r1 = 3p1 q2 q3 , N r2 = 3p2 q3 q1 , N r3 = 3p3 q1 q2 N Do phânhoạch Q = A ∪ B ∪ C Q dẫn đến phânhoạch S = A′ ∪ B ′ ∪ C ′ S với A′ = A ∩ S, B ′ = B ∩ S, C ′ = C ∩ S Dễ thấy A′ + B ′ , B ′ + C ′ , C ′ + A′ không giao đôi cách phânhoạch thỏa mãn đề Do tính nhât phânhoạch Z thỏa mãn đè nên tập A′ , B ′ , C ′ lớp đồng dư theo modulo 3, mà nhân với số N1 Mà ta dễ thấy số có dạng N3 thuộc tập tập: A′ , B ′ C ′ Mà ta lại có r1 , r2 , r3 có dạng N3 số phải thuộc tập tập A′ , B ′ C ′ Tuy nhiên 16 r1 , r2 , r3 lại thuộc tập khác A′ , B ′ , C ′ chọn từ tập khác A, B, C Điều giả sử vơ lý Vậy khơng thể có cách phânhoạch Q có tính chất thỏa mãn đề Bài tốn 12: Một cách phânhoạch π số nguyên dương n ≥ 1, định nghĩa cách biểu diễn n thành tổng hay vài số nguyên dương mà hạng tử tổng xếp theo thứ tự khơng giảm (Ví dụ, n = 4, phânhoạch π + + + 1, + + 2, + 3, + 2, 4) Gọi ℘[n] tập hợp cách phânhoạch π số nguyên dương n ≥ Với cách phânhoạch π, ta định nghĩa A(π) số số xuất π, định nghĩa B(π) số số nguyênphân biệt xuất π (Ví dụ: n = 13 π cách phânhoạch + + + + + 5, A(π) = B(π) = 3) Chứng minh với n cố định, tổng A(π) chạy khắp cách phânhoạch π n tổng B(π) chạy khắp cách phânhoạch π n hay π∈℘[n] A(π) = π∈℘[n] B(π) (1986 USAMO ) Lời giải: Ta tính giá trị π∈℘[n] B(π) trước Quả thực gọi S tập hợp {1, 2, · · · , n}, đếm số i với i ∈ S sau: n F (x) = i=1 n j=1 (1 − xj ) − n j=1,j=i (1 − xj ) x j (1 − x) n j=1 (1 − x ) Giờ ta tính giá trị π∈℘[n] A(π), ý ta có song ánh phânhoạch có i số với i ∈ S tập hợp n − i phần tử mà không chứa số hết ghép lại với ta phânhoạch n Với cách phânhoạch mà khơng có số biểu diễn sau: Rút gọn lại ta thu hệ sinh π∈℘[n] B(π) là: F (x) = f (x) = n j=2 (1 − xj ) Bây với i, ta có i số một, hệ số xn−i khai triển f (x) phải nhân với i Từ ta tính π∈℘[n] A(π) sau: n ixi f (x) i=1 Từ dẫn đến hệ sinh π∈℘[n] A(π) là: G(x) = So sánh cách hệ số F (x) G(x) ta thu §3 x f (x) (1 − x)2 π∈℘[n] A(π) = π∈℘[n] B(π) Tính chia hết Việc phân tích số thành tích thừa số nguyên tố thường dễ dàng xử lý câu hỏi chia hết có dạng sau: số chia hết cho số khác lũy thừa thừa số nguyên tố phân tích thành nhân tử số thứ lớn lũy thừa số nguyên tố phân tích thành nhân tử số thứ hai, dạng tìm ước chung lớn bội chung lớn Tuy nhiên nội dung tính chia hết khơng đơn giản đó, cần nhiều kỹ thuật chứng minh quan trọng tính chất chia hết số học Bài toán 1: Tìm số nguyên dương nhỏ K cho với tập K phần tử {1, 2, , 20} chứa phần tử phân biệt a, b cho a + b số nguyên tố Lời giải: Ta có câu trả lời 11 Ta có tập {2, 4, 6, , 20} không chứa hai phần tử phân biệt a, b thỏa mãn a + b số nguyên tố Do |{2, 4, 6, , 20}| = 10, nên K nhỏ 10.(vì K < 10 ta lấy tập {2, 4, 6, , 20} có điều mâu thuẫn) Suy K > 10 17 Ta chứng minh K ≤ 11 : Ta chia tập {1, 2, , 20} thành 10 sau {1, 2, , 20} = ({3, 20} ∪ {4, 19} ∪ · · · ∪ {11, 12}) ∪ {1, 2} Ta có tập {1, 2, , 20} có 11 phần tử có phần tử nằm (nguyên lý Dirichlet), Suy tổng số số nguyên tố (3 23) Vậy tốn có đáp số K = 11 Bài toán 2: Một tập hợp S gồm vài số nguyên dương gọi xoáy với ba số phân biệt a, b, c ∈ S, ta có a | bc, b | ca, c | ab.(Iberoamerican Olympiad 2013 ) a) Chứng minh với tập hữu hạn số nguyên dương {c1 , c2 , , cn } tập xốy tồn vơ hạn số ngun dương k cho tập {kc1 , kc2 , , kcn } tập xoáy b) Chứng minh cho số nguyên dương n ≥ tồn tập xốy gồm n phần tử thỏa mãn không tồn số nguyên dương lớn ước n phần tử Lời giải: a) Ta việc chọn sau k = m · lcm (c1 , c2 , · · · , cn ) với m ∈ Z+ n b) Xét n số nguyên tố phân biệt pi , ≤ i ≤ n, chọn ci = pj Vậy ta có pi j=1 tập hợp {c1 , c2 , , cn } thỏa mãn đề Bài tốn 3: Tìm tất số nguyên k ≥ cho tồn tập hợp M gồm k số nguyên dương cho tích số M chia hết cho tổng số phân biệt M (Czech MO 2014-2015) Lời giải: Xét k ≥ số nguyên dương phân biệt a1 , a2 , , ak Xét d = 1≤i điểm thuộc S Khi a có hữu hạn ước nguyên tố, giả sử p1 , · · · , pn Giả sử phản chứng không tồn số nguyên lớn cho ước phần tử S Giả sử với số nguyên tố pi bất kỳ, tồn phần tử S không chia hết cho pi : Khi đó, với i, ta chọn xi ∈ S thỏa mãn xi không chia hết cho pi Dễ chứng minh T = {a, x1 , · · · , xn } tập hữu hạn S thỏa mãn ước chung lớn tất phần tử T Mâu thuẫn Vậy ta có điều phải chứng minh (b) Xét p1 , · · · , pk+1 đôi nguyên tố Xét P = p1 p2 · · · pk+1 Pi = pPi Xét S tập số nguyên dương thỏa mãn chia hết cho số Pi không chia hết cho P , phần tử S chia hết cho số Pi Ta thấy S chứa tất số Pi , khơng có ước chung lớn tất phần tử S, thỏa mãn điều kiện thứ toán Tiếp theo ta xét T tập khác rỗng S có nhiều k phần tử Do phần tử S chia hết cho số Pi có k + số Pi , nên có số Pi , giả sử P1 mà khơng có số thuộc T bội Suy phần tử T chia hết cho p1 Vậy ta có điều phải chứng minh Bài tốn 5: Chứng minh với số nguyên n ≥ 2, tồn tập hợp S gồm n số nguyên cho ab chia hết cho (a − b)2 với a, b ∈ S phân biệt.(USA, 1998) Lời giải: Ta thực quy nạp theo n Xét trường hợp tầm thường n = 2, ta chọn số liên tiếp thỏa mãn đề Gọi a1 , a2 , · · · , aj j số thỏa mãn (am − an )2 | am an ∀ m = n 18 Xét M = m=n (am − an )2 · a21 a22 · · · a2j Ta nhận thấy tập {M + a1 , M + a2 , , M + aj } thỏa mãn yêu cầu toán với m = n, (am − an )2 = ((M + am ) − (M + an ))2 | am an =⇒ ((M + am ) − (M + an ))2 | (M + am )(M + an ) Ta cần thêm số khác vào tập để thu có j + phần tử thỏa mãn đề Thêm M vào tập trên, ((M + ) − M )2 = a2i | M (M + ) ∀ i Khi ta có tập gồm j + phần tử {M, M + a1 , M + a2 , , M + aj } thỏa mãn yêu cầu toán Theo nguyên lý quy nạp, ta có điều phải chứng minh Bài toán 6: Xét tập hợp S = {1, 2, , 50} Tìm số nhỏ k thỏa mãn với tập k phần tử S, tồn hai phần tử phân biệt a, b thỏa mãn ab chia hết cho a + b Lời giải: Gọi d = gcd(a, b), đặt a = dm, b = dn (m, n số nguyên dương (m, n) = 1) Điều kiện a, b viết thành: d(m + n) ước d2 mn ⇒ m + n ước dmn Hơn nữa, (m, n) = 1, suy m+n √ nguyên tố với m n, (m+n)|d Do a + b ≤ 99 ⇒ m + n ≤ 99, suy m + n ≤ Ta thấy m + n ≥ + = (a, b phân biệt), ta xét trường hợp (a, b): (6, 3); (12, 6); (18, 9); (24, 12); (30, 15); (36, 18); (42, 21); (48, 24); (12, 4); (24, 8); (36, 12); (48, 16) (20, 5), (40, 10), (15, 10), (30, 20), (45, 30) (30, 6) (42, 7), (35, 14), (28, 21) (40, 24) (45, 36) Có 23 bơ 24 số Gọi G tập 26 lại Suy ra, k > 26 (nếu không, chọn tập có phần tử thuộc G khơng thỏa mãn đề bài!) Ta cần tìm giá trị nhỏ k − 26 thỏa mãn với k − 26 chọn từ 24 số có ] + = 13 giá trị liệt kê bên Theo định lý Pigeonhole,k − 26 ≥ [ 24 nhỏ k 39 Bài tốn 7: Tìm tất tập hợp A = {a1 , a2 , , an ; n > 2012} gồm số nguyên có tính chất sau: 2012 ∈ A, đồng thời tập tùy ý gồm 2012 số thuộc A chia thành nhóm có số phần tử tổng phần tử nhóm Lời giải: Điều kiện toán cho thấy tổng 2012 số tùy ý thuộc A số chia hết cho Thay từ tập hợp 2012 phần tử phần tử phần tử khác tùy ý không thuộc tập chọn, tính chất tổng chia hết cho khơng thay đổi Như thấy rằng, phần tử thuộc A đồng dư modulo Dĩ nhiên ta nẩy dự đoán rằng, phần tử A Để kiểm chứng dự đoán này, ta xét phần tử nhỏ A a Khi tập hợp B = a1 − a, a2 − a, , an − a rõ ràng thỏa mãn tính chất nêu toán tập A Do đó, phần tử B đồng dư modulo 4, B chứa phần tử nên suy phần tử B chia hết cho Từ đây, suy luận theo quy nạp lùi quen thuộc cho ta lời giải: tập hợp nhận từ B b cách thay phần tử b ∈ B có tính chất nêu ra; phần tử tập hợp chia hết cho Tiếp tục trình, dễ suy phần tử B Như số thuộc A 2012 Bài toán 8: Với số nguyên dương d, gọi f (d) số nguyên dương nhỏ có d ước số dương Chứng minh với k ≥ 0, f (2k+1 ) chia hết cho f (2k ) Lời giải: Để giải này, rõ ràng ta cần biết số có 2k ước dương, sau tìm số nhỏ số Lẽ tự nhiên ta nghĩ đến việc phân tích số nguyên dương n thừa số ngun tố Ta có phân tích n: n = Πp pα (p) α(p) số nguyên dương 19 Gọi d(n) số ước n ta có: d(n) = Πp (α(p) + 1) Như vậy, d(n) lũy thừa với p, tồn b(p) ≥ cho α(p) = 2b(p) − Ta viết lại cấu trúc α(p) dạng sau α(p) = + 21 + + 2b(p)−1 b(p)−1 i n = Πp Πi=1 p2 Khi d(n) = 2k , với k = Σp b(p) Như số n cần tìm tích số r có dạng p2 , với p số nguyên tố r ngun khơng âm Từ phân tích ta r m thấy số p2 có mặt biểu diễn n dạng tích, số p2 với m < r có mặt tích Nghĩa số có mặt biểu diễn n ước số tham gia biểu diễn Do ta có kết luận sau r đây: gọi S tập hợp số dạng p2 với p số ngun tố r ngun khơng âm, d(n) lũy thừa n tích phần tử thuộc tập hữu hạn T S có tính chất sau: với t ∈ T, s ∈ S, mà s|t s ∈ T Hơn nữa, d(n) = 2k tập hợp T gồm k phần tử Dễ thấy với k nguyên dương, tập hợp Tk gồm k phần tử nhỏ S thỏa mãn tính chất trên, suy f (2k ) tích phần tử thuộc Tk Từ suy kết luận tốn Chú ý 3.0.2 thấy S = 2, 3, 4, 5, 7, 9, 11, 13, 16, 17, , suy f (2) = 2, f (4) = 2.3 = 6, f (8) = 2.3.4 = 24, f (16) = 2.3.4.5 = 120, f (32) = 2.3.4.5.7 = 840, Bài toán 9: Cho 81 số nguyên dương phân biệt cho ước nguyên tố chúng thuộc tập {2, 3, 5} Chứng minh tồn số 81 số mà tích chúng lũy thừa bậc số nguyên Lời giải: Ta có số nguyên dương biểu diễn dạng 2αi · 3βi · 5γi Xét đồng dư αi , βi , γi modulo Ta có αi , βi , γi có số dư khác modulo 2, có · · = dạng khác lũy thừa Theo nguyên lý Dirichle, > Ta xét tích số đặt tích a1 có số có dạng số mũ, 81 xóa số Ta tiếp tục làm thu tương tự √ = 36 Ta thấy số số lại dãy Khi ta thu 81−9 số tự nhiên số phương (Và ta lại thấy số mũ số có dạng √ √ số mũ theo modulo Theo nguyên lý Dirichle, có số am an có dạng > Xét tích am an ta lũy thừa bậc 4, số mũ theo modulo 2, 36 chúng số phương dạng số dư modulo số mũ Ta có điều phải chứng minh Chú ý 3.0.3 Ta giới hạn tốt số 25 (thay 81 bài) Bài tốn 10: Cho tập M gồm 1985 số nguyên dương phân biệt, cho ước nguyên tố 1985 số không lớn 23 Chứng minh M chứa tập gồm có phần tử mà tích chúng lũy thừa bậc số nguyên Lời giải: Cách 1: Ban đầu ta tìm số mà cho tích chúng lập thành số phương Tổng cộng ta có số nguyên tố, tối đa có 29 số có dạng mũ khác theo modulo 2, ta nhặt 29 + số tồn số có dạng mũ theo modulo rõ ràng tích chúng lập lên số phương, bỏ số dãy, ta quan tâm đến dãy 1983 số lại Tiếp tục nhiều lần ta tìm (1985 − (29 + 1))/2 ≥ 29 + cặp số mà có tích số phương Cho nên ta lập 29 + số phương, tương tự toán trước ta lại tìm số mà tích chúng lũy thừa bậc số tự nhiện Ta tập gồm phần tử mà tích chúng lũy thừa bậc bốn số nguyên Chú ý 3.0.4 Thực tế ta cần tới 1537 số nguyên dương phân biệt tập M đủ giải toán Điều chứng minh cách làm sau: 20 Cách 2: Các số có ước nguyên tố ≤ 23 có dạng n = 2a1 · 3a2 · · 23a9 , ta đồng với véc tơ v = (a1 (mod 4), , a9 (mod 4)) ⊂ Z94 , với Z4 nhóm hốn vị có bậc Ta phải chứng minh với 1985 phần tử cho Z94 , we can find mà có tổng Ta ký hiệu mệnh đề dướng Pk (n), với n số nguyên dương ≥ k số nguyên dương "Cho 2k n − (2k − 1) phần tử Zkn , ta ln tìm n số có tổng 0." Ta chứng minh dạng sau Pk (a), Pk (b) ⇒ Pk (ab) Khi đó, Pk (2) dễ chứng minh dựa vào nguyên lý Dirichlet, ta suy Pk (4) điều muốn P9 (4), điều phải chứng minh Giả sử Pk (a), Pk (b) xét 2k ab − (2k − 1) số nguyên dương Chọn nhóm có 2k a − (2k − 1) phần tử từ chọn a phần tử mà có tổng chia hết cho a Xóa a phần tử dãy ta thu dãy có 2k ab − (2k − 1) − a phần tử lại Và tiếp tục trình ta có 2k b − (2k − 1) nhóm phân biệt có a phần tử, mà tổng phần tử nhóm chia hết cho a Tiếp tục chia 2k b − (2k − 1) tổng cho a, từ nhóm có 2k b − (2k − 1) phần tử ta lấy b phần tử có tổng chia hết cho b Nhân lại tổng với a, ta thu nhóm có ab phần tử từ b tổng Rõ ràng nhóm có ab phần tử có tổng chia hết cho ab Ta kết thúc quy nạp Bài toán 11: Cho S tập hữu hạn số nguyên, số lớn Giả sử, với số nguyên n, tồn s ∈ S thỏa mãn ước chung lớn s n s Chứng minh tồn s, t ∈ S cho ước chung lớn chúng số nguyên tố (Putnam, 1999) Lời giải: Xét số nguyên dương nhỏ k thỏa mãn k không nguyên tố đôi với tất phần tử S (tồn k lấy bội chung nhỏ tất phần tử S ta tìm số nhỏ đó) Nếu tồn n ∈ S số nguyên tố ta có điều phải chứng minh cách xét gcd(n, n) Nếu không tồn n ∈ S số nguyên tố, lấy ước nguyên tố p|n cho kp số nguyên dễ thấy số nhỏ k Khi gcd(n, k) = n theo giả thiết Vì giả sử k trên, tồn n′ S mà kp nguyên tố với n′ Điều xảy vp (k) = Dễ thấy p|n′ Do gcd(n′ , k) > nên gcd(n′ , k) = p gcd(n′ , n) = p Ta có điều phải chứng minh Bài tốn 12: Chứng minh tồn tập A gồm số nguyên dương cho cho thỏa mãn tính chất sau: Với tập vô hạn S gồm số nguyên tố, tồn số nguyên dương k ≥ số nguyên dương m ∈ A n ∈ A cho m, n tích k phần tử phân biệt S (IMO, 1994) Lời giải: Tập A tập số nguyên dương xác định sau: tích n k số nguyên tố phân biệt p1 , , pk thuộc A if n ≡ (mod k + 1) Ví dụ: p1 · · pk ∈ A ⇔ p1 · · pk ≡ (mod k + 1) với k số nguyên tố pi Gọi S tập vô hãn số nguyên tố gọi p1 số nguyên tố nhỏ p2 số nguyên tố nhỏ thứ hai thuộc S Gọi p số nguyên tố thỏa mãn p > p2 Xét S modulo p Theo nguyên lý Dirichlet, số dư modulo p xuất vơ hạn lần S Chẳng hạn: tồn số nguyên a, cho tồn vô số q ∈ S mà q ≡ a (mod p) Rõ ràng, p ∤ a Chọn p − số nguyên tố phân biệt q1 , , qp−1 ∈ S đặt m = q1 · · qp−1 Khi m ∈ A theo định lý Fecma nhỏ, m = q1 · · qp−1 ≡ ap−1 ≡ (mod p) Mặt khác, ta có p > p2 > p1 , p2 ≡ p1 (mod p) Suy ra, có số số p1 · ap−2 p2 · ap−2 khơng đồng dư modulo p Suy ra, có số số n1 = p1 · q1 · · qp−2 n2 = p2 · q1 · · qp−2 không đồng dư modulo p suy khơng thuộc A Bài toán 13: Cho S = {1, 2, 3, , 280} Tìm số nguyên dương n bé cho với tập n phần tử S chứa số mà đôi nguyên tố (IMO, 1991) 21 Lời giải: Gọi A2 - tất số thuộc S,- chia hết cho 2; A3 - tất số thuộc S,- chia hết cho không thuộc A2 ; A5 - tất số thuộc S, chia hết cho không thuộc A2 , A3 ; A7 - tất số thuộc S, chia hết cho and không thuộc A2 , A3 , A5 |A2 | = 140, |A3 | = 47, |A5 | = 19, |A7 | = 10 Gọi Ap - tất số nguyên tố thuộc (7, 280), An - tất số, không thuộc A2 , A3 , A5 , A7 , Ap |Ap | = 56 Rõ ràng An = {11.11, 11.13, 11.17, 11.19, 11.23, 13.13, 13.17, 13.19}, |An | = S = A2 ∪ A3 ∪ A5 ∪ A7 ∪ Ap ∪ An Ta có tập Ax (x = 2, 3, 5, 7, p, n) khơng có phần tử chung Ta xây dựng tập A = A2 ∪ A3 ∪ A5 ∪ A7 ; |A| = 140 + 47 + 19 + 10 = 216 Lấy ngẫu nhiên số từ A Theo nguyên lý Dirichle, có hai số ta lấy từ tập Ax (x = 2, 3, 5, 7) Rõ ràng số khơng ngun tố Do n ≥ |A| + = 217 Ta chứng minh tập B - tập S |B| = 217 B chứa số ngun tố đơi Gọi P tập số chứa số nguyên tố nhỏ 280 Suy |P | = 60 Xét tập sau: A1 = {2.41, 3.37, 5.31, 7.29, 11.23, 13.19}; A2 = {2.37, 3.31, 5.29, 7.23, 11.19, 13.17}; A3 = {2.31, 3.29, 5.23, 7.19, 11.17, 13.13}; A4 = {2.29, 2.23, 5.19, 7.17, 11.13}; A5 = {2.23, 3.19, 5.17, 7.13, 11.11}; ⇒ |P ∪ A1 ∪ · · · ∪ A5 | = 88 B chứa ≥ 280 − 217 + 88 = 25 phần tử thuộc P ∪ A1 ∪ · · · ∪ A5 Nếu tồn phần tử thuộc tập P Ai với i = 1, 2, , ta thu số đôi nguyên tố Nếu không, ta chọn tối đa 4.6 = 24 phần tử Vô lý Vậy n = 217 Bài toán 14: Xét tập hợp S = {1, 2, 3, · · · , 6n}, n > Tìm giá trị lớn k cho khẳng định sau đúng: với tập A S chứa 4n phần tử có k cặp (a, b), a < b b chia hết cho a.(Brazilian 2015) Lời giải: Lemma: Cho n ∈ N gọi S tập có n + phần tử {1, 2, · · · 2n} Khi S có phần tử thỏa mãn số chia hết cho số lại Chứng minh: Phân tích {1, 2, · · · , 2n} tạo thành tập có dạng {p · 2k | k ∈ N0 } với số nguyên tố p ≤ 2n Do ta phân tích tập ban đầu thành π(2n) ≤ n tập con, theo nguyên lý Dirichle, tồn phần tử S phải thuộc tập Khi phần tử chia hết cho số lại, điều phải chứng minh Quay trở lại toán, xét A = {2n + 1, 2n + 2, · · · , 6n} Nếu x, y ∈ A số phân biệt thỏa mãn x | y, xy < 3, suy y/x = Khi đó, ta dễ chia tất (x, y) ∈ A2 thỏa mãn x | y x = y sau: có dạng (2n + t, 4n + 2t) với t = 1, 2, · · · , n Nó kéo theo k ≤ n Bây ta chứng minh k ≥ n Giả sử ngược lại, gọi (x1 , y1 ) , (x2 , y2 ) , · · · , (xm , ym ), với m < n, tất (x, y) ∈ A2 thỏa mãn x | y and x = y Ta xóa x1 , x2 , · · · , xm khỏi A tạo tập B Rõ ràng tập B khơng có (x, y) ∈ B thỏa mãn x | y and x = y Tuy nhiên, |B| ≥ |A| − m ≥ 3n + 1, mâu thuẫn với bổ đề Điều kéo theo k ≥ n , suy k = n §4 Các tốn tổng "Điều xảy tập sô nguyên cộng vào nhau?" câu hỏi nghiên cứu lớn số học-tổ hợp, kết nối tổ hợp với không số học mà với đại số giải tích Một tốn tiếng mà đến chưa giải giả thiết Goldbach mạnh "Tất số chẵn lớn biểu diễn thành tổng hai số nguyên tố" Trong mục này, tập cung cấp kỹ thuật tổ hợp đại số giá trị, điển hình nguyên lý Dirichlet, nguyên lý cực hạn, đếm nhiều cách 22 Bài toán 1: Chứng minh tập hợp n số nguyên chứa tập khác rỗng mà tổng phần tử chúng chia hết cho n Lời giải: Xét số nguyên x1 , , xn , xét tổng thành phầnx1 , x1 + x2 , x1 + x2 + x3 , , x1 + x2 + · · · + xn lập từ số Giả sử khơng có tổng chia hết cho n, phải tồn tổng có số dư Vậy hiệu chúng chia hết cho n Bài toán 2: Cho A tập {0, 1, 2, , 1997} chứa nhiều 1000 phần tử Chứng minh A chứa lũy thừa 2, chứa phần tử phân biệt cho tổng chúng lũy thừa (Ireland, 1997) Lời giải: Giả sử A không thỏa mãn yêu cầu toán Xét 996 dời nhau: tổng 2048: (1997, 51), (1996, 52), , (1025, 1023) tổng 64: (50, 14), (49, 15), (48, 16), , (33, 31) tổng to 16: (13, 3), (12, 4), (11, 5), , (9, 7) Ta chọn nhiều số từ cặp Và ta phải tránh số lũy thừa ,vì ta phải chọn số không thuộc 996 Khi ta chọn Tất lại chưa đưa vào 1, 2, 8, 32, 1024, số chọn ta có mâu thuẫn với giả sử Do tập A có nhiều 997 phần tử Ví dụ ta chọn A = {1997, 1996, , 1025, 50, 49, 48, , 33, 13, 12, 11, , 9, 0} ta có tập 997 phần tử thỏa mãn yêu cầu toán Vậy giả sử sai, ta có điều phải chứng minh Sau thi tuyển chọn vòng TST Poland với dạng câu hỏi mở sau: Bài toán 3: Cho X tập hợp {1, 2, 3, , 2001} Tìm số n nhỏ cho với tập n phần tử X ta ln tìm phần tử lũy thừa ta tìm phần tử mà tổng chúng lũy thừa (Poland TST 2001) Lời giải: Xét 998 cặp sau: (2, 14), , (7, 9) (15, 17), (18, 46), (19, 45), , (31, 33) (47, 2001), (48, 2000), , (1023, 1025) số lại 1, 8, 16, 32, 1024 Với tập S có 999 phần tử X, có số 1, 8, 16, 32, 1024 ∈ S ta có điều phải chứng minh Ngược lại, theo nguyên lý Dirichle, tồn cặp 999 tập liệt kê thuộc S Với n = 998 ta chọn tập {9, 10, , 14} ∪ {17} ∪ {33, 34, , 46} ∪ {1025, 1026, , 2001} ví dụ khơng thỏa mãn đề Vậy n = 999 giá trị nhỏ cần tìm Bài tốn 4: 50 số ngun chọn từ tập hợp {1, , 99}, cho khơng có hai số có tổng 99 100 Chứng minh số chọn phải 50, 51, 52, , 99 (St Petersburg, 1997) Lời giải: Nhóm số nguyên 1, , 100 thành 49 {x, 100 − x} with x ∈ {1, , 49}, lại số 50 Nếu ta chọn đồng thời hai số nguyên từ 49 đầu tiên, tổng chúng 100 Cho nên cặp lấy xác số Theo đầu ta cần lấy 50 số từ tập {1, , 99} nên ta buộc phải chọn số 50 Do khơng có số có tổng 99 nên từ cặp {49, 51} ta lấy số 51 Tương tự ta chọn số sau 52, 53, , 99 Ta có điều phải chứng minh Chú ý 4.0.5 Với tốn điều kiện khơng có tổng hai số 99 lỏng Ta bỏ điều kiện cho ta tốn sau : "50 số nguyên chọn từ 100 số tự nhiên cho khơng có số nguyên có tổng 100 Chứng mỉnh tồn số phương số chọn trên." Cũng với cách chia ta buộc chọn số cặp {36, 64} ta có điều phải chứng minh Bài tốn 5: Cho dãy n số nguyên dương mà tổng chúng nhỏ 2n Chứng minh 23 với số nguyên dương m không vượt tổng n số nguyên dương trên, chọn vài số nguyên cho tổng chúng m Lời giải: Ta quy ước tập rỗng có tổng Ta chứng minh quy nạp Dễ thấy trường hợp n = Giả sử toán với số nhỏ n Ta xét trường hợp sau: Trường hợp 1: số nguyên dãy Khi ta tiếp tục chọn ta đạt m, không đạt m − cơng việc lại ta chọn thêm số (trong dãy phải có số ngược lại tổng dãy 2n - mâu thuẫn với giả thiết) ta chọn hết số dãy mà tổng nhỏ m − ta chọn tiếp đến số 1’ có tổng m Trường hợp 2: tồn số nguyên k > Khi ta xóa số nguyên khỏi dãy cho ta có dãy n − số nguyên với tổng S − k số nhỏ 2n − k < 2(n − 1) Nếu m ≤ S − k, áp dụng giả thiết quy nạp ta có điều phải chứng minh Xét trường hợp S − k < m ≤ S, ta có dãy có tổng m − k (giả thiết quy nạp) ta cộng thêm số bị xóa k (Ta phải chứng minh m − k ≥ trường hợp này, nhiên điều dễ thấy từ k ≤ n S − k ≥ n − 1) Bài toán 6: Chứng minh từ tập 10 phần tử gồm số có hai chữ số, chọn hay khơng tập khác rỗng rời mà tổng phần tử chúng (IMO, 1972) Lời giải: Ta thấy tổng lớn phần tử tập có 10 phần tử 91 + 92 + + 100 = 955 Ta có 210 − = 1023 tập khác rỗng tập có 10 phần tử, suy theo nguyên lý Dirichle, ta có tập có giá trị tổng phần tử Gọi tập A B, xét giao tập A ∩ B Nếu A ∩ B = ∅, ta tập rời A B, ta có điều phải chứng minh Nếu A ∩ B = ∅, xét tập A − (A ∩ B) B − (A ∩ B) Hai tập rời nhau, chúng có tổng, nên ta có điều phải chứng minh Bài tốn 7: Tìm số ngun dương lớn n cho tồn n số nguyên không âm x1 , x2 , , xn , thỏa mãn với dãy phần tử ǫ1 , ǫ2 , , ǫn thuộc {−1, 0, 1}, không 0, n3 không ước ǫ1 x1 + ǫ2 x2 + · · · + ǫn xn (Romania, 1996) Lời giải: Ta lấy ǫi thuộc tập {0, 1} , tất số lập từ dãy ta lấy phần dư chúng chia cho n3 , trừ chúng với ta số biểu diễn ǫi ∈ {−1, 0, 1}và chia hết cho n3 tồn số dư giống Suy 2n ≤ n3 , suy ≤ n ≤ Ta cần chứng minh n = thỏa mãn yêu cầu toán Với x1 = 1, x2 = 2, x3 = 4, , x9 = 28 = 256 dễ thấy dãy thỏa mãn yêu cầu toán, ta có n = giá trị cần tìm Bài tốn 8: Cho số ngun n > Tìm số nguyên lớn d, cho từ tập S gồm n số ngun, ta tìm tập khác (khác rỗng không thiết dời nhau), thỏa mãn tổng phần tử tập chia hết cho d (Czech and Slovak, 2015) Lời giải: Xét S = {n + 1, 2n + 1, , n2 + 1} để thỏa mãn u cầu tốn ta phải có d < n, có tập khác rỗng có tổng phần tử chia hết cho n tập S Ta khẳng định d = n − 1, sử dụng bổ đề quen thuộc sau Bổ đề Với tập cho có (ít nhất) N (phân biệt) số ngun, tồn tập khác rỗng có tổng phần tử chia hết cho N Đây kết qu Erdă os ó bit B Ch cú nhât tập gồm N − (phân biệt) số ngun, mà khơng có tập khác rỗng chia hết cho N , số nguyên đồng dư theo modulo N , nguyên tố với N Bổ đề quen thuộc, với lời giải sơ cấp Theo bổ đề 1, tồn tập ∅ = A ⊆ S có tổng phần tử chia hết cho n − Lấy tùy ý a ∈ A Lại theo bổ đề 1, tồn tập ∅ = B ⊆ S \ {a} có tổng phần tử chia hết cho n − 1, rõ ràng B = A Lấy tùy ý b ∈ B, suy b = a Nếu tồn tập ∅ = C ⊆ S \ {a, b} có tổng phần tử chia hết cho n − 1, ta có điều phải chứng minh Ngược lại, theo bổ đề 2, tồn r nguyên tố với n − thỏa 24 mãn x ≡ r (mod n − 1) với x ∈ S \ {a, b} Nếu a ≡ (mod n − 1) b ≡ (mod n − 1) ta dễ có điều phải chứng minh, ta cần xét trường hợp lại Nhận xét giải trường hợp n = 3, ta giả sử n > Có B tập tạo b số phần tử S \ {a, b} Nếu tất phần tử S \ {a, b} sử dụng, ta dễ có tập B ′ cách đổi số phần tử (sao cho B ′ khác B A) có tổng phần tử chia hết cho n − Chúng ta trường hợp B = S \ {a}, với b ≡ r (mod n − 1) Do A tập tạo a số khác n − phần tử lại, mà phần tử đồng dư với r modulo n − Do tất phần tử sử dụng nên ta đổi phần tử đê thu tập A′ A′′ (khác với A) có tổng phần tử chia hết cho n − Bài toán 9: Cho x1 , x2 , , x19 số nguyên dương nhỏ 93 Cho y1 , y2 , , y93 số nguyên dương nhỏ 19 Chứng minh tồn tổng vài (khác rỗng) xi tổng vài yj (Putnam, 1993) Lời giải: Giả sử mâu thuẫn Khơng tính tổng qt, giả sử i yi > i xi Với tổng thành phần số x, xét tổng thành phần số y có tổng lớn mà có số phần tử Sự khác y-tổng vàx-tổng thuộc {1, , 18} (18 ta giả sử khơng thể có tổng nhau.) Nhưng có 19 tổng thành phần j4 x, lặp lại Giả sử ji=1 xi = ji=1 yi ji=1 xi = i=1 yi ( giả sử j3 j4 j1 < j3 , j2 < j4 ) Từ suy j1 +1 xi = j2 +1 yi Điều phải chứng minh Bài tốn 10: Hỏi có tập tập hợp {1, 2, 3, , 2000} mà tổng phần tử chia hết cho 5? 2001000 k Lời giải: Xét f (x) = 2000 aj xj = + x + x2 + 2x3 + · · · + x2001000 Khi k=1 (1 + x ) = j=0 tổng cần tính a0 + a5 + · · · + a2001000 (bao gồm tập ∅) Ta cần tính f (ω m ) với ω nghiệm phương trình x5 = m ∈ {0, 1, 2, 3, 4} Đặt A0 = a0 +a5 +· · ·+a2001000 , A1 = a1 +a6 +· · ·+a2000996 , A2 = a2 +a7 +· · ·+a2000997 , A3 = a3 + a8 + · · · + a2000998 , A4 = a4 + a9 + · · · + a2000999 Khi 2400 = f (ω) = A0 + A1 ω + A2 ω + A3 ω + A4 ω , 2400 = f (ω ) = A0 + A1 ω + A2 ω + A3 ω + A4 ω , 2400 = f (ω ) = A0 + A1 ω + A2 ω + A3 ω + A4 ω , 2400 = f (ω ) = A0 +A1 ω +A2 ω +A3 ω +A4 ω, 22000 = f (1) = A0 +A1 +A2 +A3 +A4 2402 4799 + Cộng tất biểu thức ta A0 = Bài toán 11: Cho p số nguyên tố lẻ Xác định số tập có p phần tử tập hợp {1, 2, , 2p} cho tổng phần tử chia hết cho p (IMO, 1995) Lời giải: Đặt X := {1, 2, , p}, Y := {p + 1, p + 2, , 2p} Ta thấy X Y hệ đại diện Z/pZ Với tập A ⊂ X z ∈ Z/pZ đặt A + z := {a + z ∈ X|a ∈ A} tương tự với tập B ⊂ Y Gọi A ⊂ X nghiệm tầm thường A = ∅ or A = X, tương ứng với B ⊂Y Ta có tập P ⊂ (X ∪ Y ) thỏa mãn đồng thời P ∩ X P ∩ Y tầm thường, gọi tập tầm thường Khi với P ⊂ (X ∪ Y ) ta có phép tịnh tiến sau: P ∩ X khơng phải tập tầm thường P + z := ((P ∩ X) + z) ∪ (P ∩ Y ) P + z := (P ∩ X) ∪ ((P ∩ Y ) + z) trường hợp ngược lại Dễ thấy z chạy khắp hệ thặng dự modulo p, tồn xác tổng số P + z chia hết cho p Do phép tình tiến chia tập không tầm thường thành lớp tương đương, lớp có p tập, tất các tập lớp có phần tử giống Vì ta có 2p tập có sơ phần tử p số chúng nghiệm tầm thường p −2 (2p ) nên ta có p p + tập thỏa mãn Bài toán 12: Cho 2p − số nguyên với p số nguyên tố , chứng minh ta chọn chinh xác p số số cho tổng chúng chia hết cho p 25 Lời giải: Trước hết ta cần đề cập đến định lý Chevalley-Warning phát biểu sau: Chevalley - Warning theorem Xét đa thức f1 , , fm với biến x1 , , xn m hữu hạn trường Fq , với q = pk Giả sử n > i=1 deg fi Khi số nghiệm (x1 , , xn ) hệ phương trình f1 (x1 , , xn ) = 0, ,fm (x1 , , xn ) = chia hết cho p Giải bổ đề Đầu tiên, ta tìm cách đẹp để đếm số nghiệm ( mod p): q−1 Gọi g(x1 , , xn ) = m ) i=1 (1 − fi (x1 , , xn ) Ta có nhận xét: * Nếu (x1 , , xn ) nghiệm, g(x1 , , xn ) = * Nếu (x1 , , xn ) khơng nghiệm, g(x1 , , xn ) = Nhận xét dễ thấy Bây ta xét nhận xét thứ hai Do (x1 , , xn ) không nghiệm, ta có fi (x1 , , xn ) = với i Suy fi (x1 , , xn )q−1 = ta thấy tích g(x1 , , xn ) 0, suy ta chứng minh nhận xét Suy ta đếm số nghiệm mod p có đánh giá (x1 , ,xn )∈Fn g(x1 , , xn ) q Bây ta xét mở rộng g tổng đơn thức: = (x1 , ,xn )∈Fn g(x1 , , xn ) (x1 , ,xn )∈Fn q = (x1 , ,xn )∈Fn q xd11 · · · xdnn = cd1 , ,dn xd11 · · · xdnn xd11 · · · xdnn (x1 , ,xn )∈Fn q xd11 · · · xdnn = (x1 , ,xn )∈Fn q n di d = for i=1 xi ∈Fq xi Suy x∈Fq x q từ đồng thức x − x = cho x ∈ Fq (d1 , ,dn )∈Nn Nhận xét: Nếu tồn di < q − 1, Thật vậy, (d1 , ,dn )∈Nn q cd1 , ,dn d < q − Nó quen thuộc, suy từ kéo theo tất đa thức hoán vị phần tử Fq bậc lớn q − tất đa thức hốn vị bậc lớn q − Suy tổng (d1 , ,dn )∈Nn cd1 , ,dn (x1 , ,xn )∈Fn xd11 · · · xdnn tổng hạng tử q mà d1 , , dn ≥ q − Cho nên bậc hạng tử n(q − 1) Mà bậc g nhiều m i=1 (q − 1) deg fi < n(q − 1) theo giả sử Cho nên cd1 , ,dn = với tất hạng tử chứng minh xong bổ đề Xét hệ sau: + + xp−1 + xp−1 xp−1 2p−1 ≡ p−1 ≡0 + + a2p−1 x2p−1 + a2 xp−1 mod pa1 xp−1 mod p gồm phương trình theo mod p có 2p − ẩn Do tổng bậc hai phương trình (p − 1) + (p − 1) = 2p − 2, số biến nên ta áp dụng Chevalley-Warning Dễ thấy ta có nghiệm x1 = x2 = = x2p−1 = 0, số nghiệm chúng chia hết cho p > 1, nên chắn nghiệm khác Với nghiệm khác khác nghiệm x1 = x2 = = x2p−1 = 0, ta đặt yi = xip−1 Theo định lý Fermat, yi ≡ mod p ⇐⇒ xi ≡ mod p yi ≡ mod p trường hợp lại Cho nên thấy yi ≡ 0, mod p Theo giả sử yi không đồng thời 0, dựa vào phương trình thứ hệ y1 + y2 + + y2p−1 ≡ mod p cho xác p yi ≡ mod p (còn lại 0) Thay chúng vào phương trình thứ hai hệ a1 y1 + a2 y2 + + a2p−1 y2p−1 ≡ mod p cho ta thấy tổng xác p số ≡ mod p, chia hết cho p Ta có điều phải chứng minh Bài toán 13: Cho 2n − số nguyên , chứng minh ta chọn chinh xác n số số cho tổng chúng chia hết cho n (định lý Erd˝ os-Ginzburg-Ziv ) Lời giải: Gọi EGZn nghĩa mệnh đề đề với n, nghĩa ta chọn từ tập 2n − (hoặc hơn) số nguyên xác n số thỏa mãn tổng chúng chia hết cho n Khi có EGZa EGZb , ta chứng minh EGZab Chứng minh: Lấy 2ab − số cho Do ta giả sử EGZa đúng, ta chọn a số chúng thỏa 26 mãn tổng chúng s1 chia hết cho a Khi 2a(b − 1) − số lại, ta chọn lại a số thỏa mãn tổng chúng s2 chia hết cho a Tiếp tục lại a − số Ta tính ta làm 2b − lần a số cho ta s1 , s2 , , s2b−1 tổng Bây ta có 2b − tổng, ta áp dụng EGZb tìm b số si thỏa mãn tổng chúng S chia hết cho b Quay lại thay tổng si số ban đầu, ta có S tổng ab số ban đầu Suy tổng chia hết cho ab Điều phải chứng minh 27 Tài liệu [1] Hà Huy Khoái Một số toán Số học - Tổ hợp Addison-Wesley, Reading, Massachusetts, 1993 [2] Herbert S Wilf Lectures on Integer Partitions University of Pennsylvania(2000) [3] G H Hardy and S Ramanujan Asymptotic formula in combinatory analysis London Math Soc 17 (1918), 175–115 [4] Hans Rademacher On the partition function p(n) Proc London Math Soc 43 (1937), 241–254 [5] Art of problem solving Forum, http://www.artofproblemsolving.com/ [6] Diễn đàn toán học, http://diendantoanhoc.net/ 28 ... B, giả sử số x1 , x2 , x3 , x4 , x5 Năm số biểu diễn 10 số số từ 15 đến 24, tức 10 số 10 tổng {xk +xl , k = l; ≤ k ≤ 15} Từ suy 15 + 16 + + 24 = 4(x1 + x2 + x3 + x4 + x5 ) số xi tham gia cặp... (a, b): (6, 3); (12, 6); (18, 9); (24, 12); (30, 15) ; (36, 18); (42, 21); (48, 24); (12, 4); (24, 8); (36, 12); (48, 16) (20, 5), (40, 10), (15, 10), (30, 20), (45, 30) (30, 6) (42, 7), (35,... nguyên phân hoạch thành dãy cấp số cộng rời Chứng minh tồn hai dãy cấp số cộng mà có cơng sai (Sasha Schwartz) Lời giải: Giả sử tập số nguyên dương bị chia thành hữu hạn tập S1 , S2 , , Sk , tập